Difference between revisions of "2002 AMC 12A Problems/Problem 2"

(New page: ==Problem== {{empty}} ==Solution== {{solution}} ==See Also== {{AMC12 box|year=2002|ab=A|num-b=1|num-a=3}})
 
Line 1: Line 1:
 
==Problem==
 
==Problem==
  
{{empty}}
+
Cindy was asked by her teacher to subtract 3 from a certain number and then divide the result by 9. Instead, she subtracted 9 and then divided the result by 3, giving an answer of 43. What would her answer have been had she worked the problem correctly?
 +
 
 +
<math> \mathrm{(A) \ } 15\qquad \mathrm{(B) \ } 34\qquad \mathrm{(C) \ } 43\qquad \mathrm{(D) \ } 51\qquad \mathrm{(E) \ } 138 </math>
 +
 
  
 
==Solution==
 
==Solution==
  
{{solution}}
+
<math>\dfrac{x-9}{3}=43</math>
 +
 
 +
<math>x=43*3+9=138</math>
 +
 
 +
<math>\dfrac{x-3}{9}=15 \Rightarrow \mathrm {(A)}</math>
  
 
==See Also==
 
==See Also==
  
 
{{AMC12 box|year=2002|ab=A|num-b=1|num-a=3}}
 
{{AMC12 box|year=2002|ab=A|num-b=1|num-a=3}}

Revision as of 14:35, 17 October 2007

Problem

Cindy was asked by her teacher to subtract 3 from a certain number and then divide the result by 9. Instead, she subtracted 9 and then divided the result by 3, giving an answer of 43. What would her answer have been had she worked the problem correctly?

$\mathrm{(A) \ } 15\qquad \mathrm{(B) \ } 34\qquad \mathrm{(C) \ } 43\qquad \mathrm{(D) \ } 51\qquad \mathrm{(E) \ } 138$


Solution

$\dfrac{x-9}{3}=43$

$x=43*3+9=138$

$\dfrac{x-3}{9}=15 \Rightarrow \mathrm {(A)}$

See Also

2002 AMC 12A (ProblemsAnswer KeyResources)
Preceded by
Problem 1
Followed by
Problem 3
1 2 3 4 5 6 7 8 9 10 11 12 13 14 15 16 17 18 19 20 21 22 23 24 25
All AMC 12 Problems and Solutions